Difference between revisions of "1975 AHSME Problems/Problem 27"
Megaboy6679 (talk | contribs) (→Solution 1) |
(fix mistake) |
||
(2 intermediate revisions by 2 users not shown) | |||
Line 12: | Line 12: | ||
<cmath>p^3 + q^3 + r^3 = (p^2 + q^2 + r^2) - (p + q + r) + 6.</cmath> | <cmath>p^3 + q^3 + r^3 = (p^2 + q^2 + r^2) - (p + q + r) + 6.</cmath> | ||
− | By Vieta's formulas, <math>p + q + r = 1</math>, <math>pq + pr + qr = 1</math>, and <math>pqr = 2</math>. Squaring the equation <math>p + q + r = 1</math>, we get | + | By [[Vieta's formulas]], <math>p + q + r = 1</math>, <math>pq + pr + qr = 1</math>, and <math>pqr = 2</math>. Squaring the equation <math>p + q + r = 1</math>, we get |
<cmath>p^2 + q^2 + r^2 + 2pq + 2pr + 2qr = 1.</cmath> | <cmath>p^2 + q^2 + r^2 + 2pq + 2pr + 2qr = 1.</cmath> | ||
Subtracting <math>2pq + 2pr + 2qr = 2</math>, we get | Subtracting <math>2pq + 2pr + 2qr = 2</math>, we get | ||
Line 24: | Line 24: | ||
~pfalcon | ~pfalcon | ||
+ | |||
+ | ==Solution 3 (Beginner's Solution)== | ||
+ | Use Vieta's formulas to get <math>p+q+r=1</math>, <math>pq+qr+pr=1</math>, and <math>pqr=2</math>. | ||
+ | |||
+ | Square <math>p+q+r=1</math>, and get <math>p^2+q^2+r^2+2pq+2pr+2qr=1</math> | ||
+ | |||
+ | Substitute <math>pq+qr+pr=1</math> and simplify to get <math>-1=p^2+q^2+r^2</math> | ||
+ | |||
+ | After that, multiply both sides by <math>1=p+q+r</math>, to get | ||
+ | <math>-1=p^3+q^3+r^3+p^2q+q^2r+p^2r+q^2r+r^2p+r^2q</math> | ||
+ | |||
+ | Then, factor out <math>pq</math>, <math>qr</math>, and <math>pr</math>: | ||
+ | <math>-1=p^3+q^3+r^3+pq(p+q)+qr(q+r)+pr(p+r)</math> | ||
+ | |||
+ | Then, substitute the first equation into <math>p+q</math>, <math>q+r</math>, and <math>p+r</math>. | ||
+ | <math>-1=p^3+q^3+r^3+pq(1-r)+qr(1-p)+pr(1-q)</math> | ||
+ | |||
+ | Then, multiply it out: | ||
+ | <math>-1=p^3+q^3+r^3+pq+qr+pr-3pqr</math> | ||
+ | |||
+ | After that, substitute the equations <math>pq+qr+pr=1</math> and <math>pqr=2</math>: | ||
+ | <math>-1=p^3+q^3+r^3+1-6</math> | ||
+ | |||
+ | Solving that, you get <math>p^3+q^3+r^3=\boxed{\text{(E) }4}</math> | ||
+ | ~EZ PZ Ms.Lemon SQUEEZY |
Latest revision as of 17:58, 23 August 2023
Problem
If and are distinct roots of , then equals
Solution 1
If is a root of , then , or Similarly, , and , so
By Vieta's formulas, , , and . Squaring the equation , we get Subtracting , we get
Therefore, . The answer is .
Solution 2(Faster)
We know that . By Vieta's formulas, ,, and . So if we can find , we are done. Notice that , so , which means that
~pfalcon
Solution 3 (Beginner's Solution)
Use Vieta's formulas to get , , and .
Square , and get
Substitute and simplify to get
After that, multiply both sides by , to get
Then, factor out , , and :
Then, substitute the first equation into , , and .
Then, multiply it out:
After that, substitute the equations and :
Solving that, you get ~EZ PZ Ms.Lemon SQUEEZY